Preview only show first 10 pages with watermark. For full document please download

Meh_eng

###

   EMBED


Share

Transcript

          This booklet is a sequel to a similar col- lection of problems on kinematics. Sim- ilarly to that collection the aim here is to present the most important ideas us- ing which one can solve most ( >  95%) of  olympiad problems on mechanics. Usu- ally a problem is stated first, and is fol- lowed by some relevant ideas and sug- gestions (letter ‘K’ in front of the number of an idea refers to the correspondingly numbered idea in the kinematics book- let). The answers to the problems are lis- tedattheendofthebooklet. Theyarepre- cededbyquitedetailedhints(nofullsolu- tions), but think carefully before reading the hints as a last resort! The guiding principle of this booklet argues that almost all olympiad problems are “variations” on a specific set of top- ics — the solutions follow from corres- ponding solution ideas. Usually it is not very hard to recognize the right idea for a given problem, having studied enough solution ideas. Discovering all the ne- cessary ideas during the actual solving would certainly show much more creativ- ity and offer a greater joy, but the skill of  conceiving ideas is unfortunately difficult (or even impracticable) to learn or teach. Moreover, it may take a long time to reach a new idea, and those relying on trying it during an olympiad would be in disad- vantage in comparison to those who have mastered the ideas. In science as a whole, solution ideas play a similar role as in olympiads: most scientific papers apply and combine known ideas for solving new (or worse, old) problems, at best developing and generalising the ideas. Genuinely new good ideas occur extremely rarely and many of them are later known as master- pieces of science. However, as the whole repertoire of scientific ideas encompasses immensely more than mere mechanics, it is not so easy to remember and utilise them in right places. The respective skill is highly valued; an especial achievement would be employing a well-known idea in an unconventional (unexpected, novel) situation. In addition to ideas, the booklet also presents “facts” and “methods”. The dis- tinction is largely arbitrary, some facts could have been called methods and vice versa. In principle, an “idea” should have wider and/or more creative applications than a “fact”; a “method” is a universal and conventionalized “idea”. Several sources have been used for the problems: Estonian olympiads’ regional and national rounds, journal “Kvant”, Russian and Soviet Union’s olympi- ads; some problems have been modified (either easier or tougher), some are “folk- lore” (origins unknown).   For problems on statics the solution is usually standard: we have to write down the condition of force balance for the  x -,  y - and (if necessary)  z -components; often the condition of torque balance must be added. Usually the main ingenuity lies in IDEA  1:              for instance, the reaction force between two bodies or the tensile force in a string (or a rod). To zero as many forces as pos- sible it is worthwhile to note that  a)  the axes may not be perpendicular;  b)  if the system consists of several bodies, then a different set of axes may be chosen for each body. IDEA  2:            For example, if we choose the pivot to  be at the contact point of two bodies, thenthemomentarmsofthefrictionforce  between the bodies and of their reaction force are both zero. IDEA  3:           x    y    An equation for the torques can be writ- ten about any pivot point (“axis” of rota- tion). In principle, we could write several equations for several pivots at the same time, but together with the equations for the forces  the maximum number of linearly independent equations equals the number of  degrees of freedom of the body  (three in the two-dimensional case, as the body can rotate in a plane and shift along the  x - and  y -axis). Accordingly, all is fine if we writeoneforcesequationandtwotorques equations (or just three torques equations — as long as the pivots do not lie on a straight line); on the other hand, if we wrote two equations of both types, then one of the four equations would always  be a redundant consequence of the three others and needless to write down. So, an equation for the force balance may be replaced by an equation for the torque balance about an additional pivot. Such a substitution may turn out to be useful if the unwanted (uninteresting) forces are unparallel, because a choice of  a projection axis can zero only one force in the balance of forces, while a choice of  a pivot for the torques can zero two forces at once. PROB  1.  An end of a light wire rod is  bent into a hoop of radius  r . The straight part of the rod has length  l ; a ball of mass  M  is attached to the other end of the rod. The pendulum thus formed is hung by the hoop onto a revolving shaft. The coef- ficient of friction between the shaft and the hoop is  µ . Find the equilibrium angle  between the rod and the vertical. r     l         M  ω µ Here we mainly need idea 2 with some simplification offered by FACT  1:     α   tan α  =  µ  PROB  2.  On an incline with slope angle α  thereliesacylinderwithmass  M , itsaxis  being horizontal. A small block with mass m  isplacedinsideit. Thecoefficientoffric- tion between the block and the cylinder is µ ; the incline is nonslippery. What is the maximum slope angle  α  for the cylinder to stay at rest? The block is much smaller than the radius of the cylinder. α m M  Here we can again use fact 1 and idea 2 if  we add IDEA  4:        Further, the net force (or torque) is the sum of forces (torques) acting on the con- stituents (the effort is eased as the internal forces are needless — they cancel each other out). In our case, it is useful to as- semble such a whole system from the cyl- inder and the block. PROB  3.  Three identical rods are connec- ted by hinges to each other, the outmost ones are hinged to a ceiling at points  A and  B . The distance between these points is twice the length of a rod. A weight of  mass  m  is hanged onto hinge  C . At least how strong a force onto hinge  D  is neces- sarytokeepthesystemstationarywiththe rod  CD  horizontal? B m A C   D F      Again we can use idea 2. The work is also aided by FACT  2:            Indeed, the net external force  ⃗  F  onto either point of application of the forces must point along the rod, as its torque with respect to the other point of applic- ation must be zero. In addition to the ex- ternal forces, the point is acted on by ten- sionforce ⃗  T   thatmustcompensatetherest of the forces, so ⃗  F  = − ⃗  T  . Some ideas are very universal, espe- cially the mathematical ones. IDEA  K5:    for example, the shortest path from a point to a plane is perpendicular to it. PROB  4.  What is the minimum force needed to dislodge a block of mass  m  rest- ing on an inclined plane of slope angle  α , if the coefficient of friction is  µ ? Investig- ate the cases when  a)  α  =  0;  b)  0  <  α  < arctan µ . α F  m  IDEA  5:      Fact 1, or rather its following generalisa- tion, turns out to be of use: FACT  3:         arctan µ  Thisfactisalsobeneficialinthenextprob- lem. PROB  5.  Ablockrestsonaninclinedsur- facewithslopeangle α . Thesurfacemoves with a horizontal acceleration  a  which lies in the same vertical plane as a normal vec- tor to the surface. Determine the values of  the coefficient of friction  µ  that allow the  block to remain still. α a  µ  Here we are helped by the very universal IDEA  6:      To clarify: in a translationally moving ref- erence frame we can re-establish New- ton’s laws by imagining that every body with mass  m  is additionally acted on by an inertial force  − m ⃗  a  where ⃗  a  is the ac- celeration of the frame of reference. Note that that the fictitious force is totally ana- logous to the gravitational force and (as an aside) their equivalence is the corner- stone of the theory of general relativity (more specifically, it assumes the inertial and gravitational forces to be indistin- guishable in any local measurement). IDEA  7:      PROB  6.  A cylinder with radius  R  spins around its axis with an angular speed ω . On its inner surface there lies a small  block; the coefficient of friction between the block and the inner surface of the cyl- inder is  µ . Find the values of   ω  for which the block does not slip (stays still with re- spect to the cylinder). Consider the cases where  (a)  the axis of the cylinder is hori- zontal;  (b)  the axis is inclined by angle  α with respect to the horizon. ω α IDEA  8:     m ω 2 ⃗  R   ω     R                         Warning:  in this idea, the axis of rotation must be actual, not instantaneous.  For the last problem, recall idea K5 and fact 3; for part  (b) , add IDEA  9:            PROB  7.  A hollow cylinder with mass  m and radius  R  stands on a horizontal sur- face with its smooth flat end in contact the surface everywhere. A thread has been wound around it and its free end is pulled with velocity  v  in parallel to the thread. Find the speed of the cylinder. Consider two cases:  (a)  the coefficient of friction  between the surface and the cylinder is zero everywhere except for a thin straight  band (much thinner than the radius of the cylinder) with a coefficient of friction of  µ , the band is parallel to the thread and its distance to the thread  a  <  2 R  (the fig- ure shows a top-down view);  (b)  the coef- ficient of friction is  µ  everywhere.  Hint: any planar motion of a rigid body can  be viewed as rotation around an instant centre of rotation, i.e. the velocity vector of any point of the body is the same as if  the instant centre were the real axis of ro- tation. v      a µ This is quite a hard problem. It is use- ful to note IDEA  10:      Also remember ideas 1 and 2. The latter can be replaced with its consequence, FACT  4:         Another useful fact is FACT  5:        From time to time some mathematical tricks are also of use; here it is the prop- ertyofinscribed angles(Thales’theorem), FACT  6:        The property of inscribed angles is also useful in the next problem, if we add (somewhat trivial) IDEA  11:   PROB  8.  A light wire is bent into a right angle and a heavy ball is attached to the  bend. The wire is placed onto supports with height difference  h  and horizontal distance  a . Find the position of the wire in its equilibrium. Express the position as the angle between the bisector of the right angle and the vertical. Neglect any fric- tion between the wire and the supports; the supports have little grooves keeping all motion in the plane of the wire and the figure. α a         h PROB  9.  A rod with length  l  is hinged to a ceiling with height  h  <  l . Under- neath, a board is being dragged on the floor. The rod is meant to block the move- ment the board in one direction while al- lowing it move in the opposite direction. What condition should be fulfilled for it to do its job? The coefficient of friction is µ 1  between the board and the rod, and  µ 2  between the board and the floor. α µ 1 µ 2 Let’s remember fact 3: if the relative sliding between two bodies has a known direction, then the direction of the sum of  the friction and reaction force vectors is always uniquely determined by the coef- ficient of friction. If a force makes one of  the bodies move in such a way that the reaction force grows, then they jam: the larger the forces we try to drag the bod- ies with, the larger friction and reaction forces restrain them. IDEA  12:                PROB  10.  Four long and four half as long rods are hinged to each other form- ing three identical rhombi. One end of  the contraption is hinged to a ceiling, the other one is attached to a weight of  mass  m . The hinge next to the weight is connected to the hinge above by a string. Find the tension force in the string. m This problem is the easiest to solve us- ing  the method of virtual displacement . METHOD  1:         ∆ x   T  ∆ x    ∆Π     T   = ∆Π / ∆ x  Generalisation:  if some additional external forces  ⃗  F i  ( i  =  1,2,...) act on the sys- temwiththedisplacementsoftheirpoints of action being  δ ⃗  x i , while the interest- ing string or rod undergoes a virtual lengthening of   ∆ x , then  T   = ( ∆Π  − ∑  i  δ ⃗  x i · ⃗  F i ) / ∆ x . The method can also be used for find- ing some other forces than tension (for example, in problems about pulleys): by imaginarily shifting the point of action of  the unknown force one can find the pro-  jection of this force onto the direction of  the virtual displacement. PROB  11.  A rope with mass  m  is hung from the ceiling by its both ends and a weight with mass  M  is attached to its centre. The tangent to the rope at its either end forms angle  α  with the ceiling. What is the angle  β  between the tangents to the rope at the weight? α  α β  m M  FACT  7:      In addition, we can employ IDEA  13:      In fact, here we do not need the idea as a whole, but, rather, its consequence, FACT  8:    In problems about ropes one may some- times use IDEA  14:          This allows us to write down the con- dition of torque balance for the hanging portion of the rope (as we know the ho- rizontal coordinate of its centre of mass). The next problem illustrates that ap- proach. PROB  12.  A boy is dragging a rope with length  L  =  50m along a horizontal ground with a coefficient of friction of   µ  = 0.6, holding an end of the rope at height  H   =  1m from the ground. What is the length  l  of the part of the rope not touch- ing the ground? PROB  13.  A light rod with length  l  is hinged in such a way that the hinge folds in one plane only. The hinge is spun with angular speed  ω  around a vertical axis. A small ball is fixed to the other end of the rod.  (a)  Find the angular speeds for which the vertical orientation is stable.  (b)  The  ball is now attached to another hinge and, in turn, to another identical rod; the upper hinge is spun in the same way. What is now the condition of stability for the ver- tical orientation? ω  ω a) b)                  l                  l                  l For answering about the stability of  an equilibrium, usually the following fact works best. IDEA  15:       ∆ x   ∆ ϕ          Incidentally use all formulae of approx- imate calculation known from mathemat- ics (sin  ϕ ≈  ϕ  and others); IDEA  16:  f  ( x  + ∆ x )  ≈  f  ( x ) +  f  ′ ( x ) ∆ x [+  f  ′′ ( x ) ∆ x 2 2  ]   ( x  + ∆ x )(  y  + ∆  y )  ≈  xy  + x ∆  y  +  y ∆ x      The case  (b)  is substantially more diffi- cult as the system has two degrees of free- dom (for example, the deviation angles ∆ ϕ 1  and ∆ ϕ 2  of the rods). Although idea 15 is generalisable for more than one de- grees of freedom, apparently it is easier to start from idea 11. IDEA  17:    x  =  y  =  0       Π ( x ,  y )     Π ( x , kx )    k   PROB  14.  If a beam with square cross- section and very low density is placed in water, it will turn one pair of its long op- posite faces horizontal. This orientation, however, becomes unstable as we increase its density. Find the critical density when this transition occurs. The density of wa- ter is  ρ v  =  1000kg/m 3 . IDEA  18:          Indeed, consider a body with density of  the liquid and shape identical to the part of the given body that is immersed in the liquid. Of course it must be in equi- librium when placed in water: whatever point we choose to measure torques from, the sum of moments from pressure forces is always equal to the opposite value of  torquefromgravity. Whencalculatingthe moments from buoyancy in this question, it is useful to keep in mind that we can give negative mass to bits of some body: if two bits overlap that have the same density with different signs, they add up to zero density. The last suggestion can be formulated in a more general way: IDEA  19:              This quantity can be mass density (like in this case), charge or current density, some force field etc. Often this trick can be com-  bined with IDEA  20:    This goal can be reached by applying idea 19, but also by using appropriate reference frames, dividing the process of  solving into several phases (where some phases use symmetric geometry), etc. PROB  15.  A hemispherical container is placed upside down on a smooth hori- zontal surface. Through a small hole at the bottom of the container, water is then poured in. Exactly when the con- tainer gets full, water starts leaking from  between the table and the edge of the con- tainer. Find the mass of the container if water has density  ρ  and radius of the hemisphere is  R . M  R IDEA  21:             The latter is given by  pS , where  p  is pres- sure of the liquid near the tabletop and  S is area of the container’s open side. PROB  16.  A block is situated on a slope with angle  α , the coefficient of friction  between them is  µ  >  tan α . The slope is rapidly driven back and forth in a way that its velocity vector ⃗  u  is parallel to both the slope and the horizontal and has con- stant modulus  v ; the direction of   ⃗  u  re- verses abruptly after each time interval  τ  . What will be the average velocity  w  of the  block’s motion? Assume that  g τ   ≪ v . x y z u t v − v  τ   u α IDEA  22:       ⟨ X  ⟩       ˜ X      X   = ⟨ X  ⟩ +  ˜ X   METHOD  2:                         In this particular case, the choice of zeroth approximation needs some explanation. The condition  g τ   ≪ v  implies that within one period, the block’s velocity cannot change much. Thereforeif the block is ini- tially slipping downwards at some velo- city  w  and we investigate a short enough time interval, then we can take the block’s velocity to be constant in zeroth approx- imation, so that it is moving in a straight line. We can then move on to phase two and find the average value of frictional force, based on the motion obtained in phase one. PROB  17.  Let us investigate the extent to which an iron deposit can influence water level. Consider an iron deposit at the bot- tom of the ocean at depth  h  =  2km. To simplify our analysis, let us assume that it is a spherical volume with radius 1km with density greater from the surround- ing rock by  ∆  ρ  =  1000kg/m 3 . Presume that this sphere touches the bottom of the ocean with its top, i.e. that its centre is situated at depth  r  +  h . By how much is the water level directly above the iron deposit different from the average water level?       IDEA  23:        If this was not the case, the potential en- ergy of the liquid could be decreased by allowing some particles on the surface to flow along the surface to where their po- tential energy is smaller. IDEA  24:      The principle of superposition still holds and a sphere’s potential only has a dif- ferent factor: instead of   Q /4 πε 0 r  in elec- trostatics the gravitational potential of a sphere with respect to infinity is  ϕ  = − GM / r ; the minus sign comes from the fact that masses with the same sign [“+”] attract. PROB  18.  A horizontal platform rotates around a vertical axis at angular velo- city  ω . A disk with radius  R  can freely rotate and move up and down along a slippery vertical axle situated at distance d  >  R  from the platform’s axis. The disk is pressed against the rotating platform due to gravity, the coefficient of friction  between them is  µ . Find the angular ve- locity acquired by the disk. Assume that pressure is distributed evenly over the en- tire base of the disk. d R   ω r   IDEA  25:          Thus  ⃗  ω 3  =  ⃗  ω 1  + ⃗  ω 2 , where  ⃗  ω 1  is angu- lar velocity of the reference frame, ⃗  ω 2  an- gular velocity of the body in the rotating frame of reference and  ⃗  ω 3  that in the sta- tionary frame. In this question, we can use fact 5, ideas 2, 8, 10 and also IDEA  K5:        METHOD  3:        Within an infinitesimal bit (period), quantities changing in space (time) can be taken constant (in our case, that quantity is the direction of frictional force vector). If necessary (see the next question), these quantities may be summed over all bits — this is called integration. PROB  19.  A waxing machine consists of a heavy disk with mass  M  densely covered with short bristles on one side, so that if it lies on the floor, then its weight is evenly distributed over a circular area with radius  R . An electrical motor makes the disk rotate at angular velocity  ω , the user compensates for the torque from fric- tional forces by a long handle. The same handle can be used to push the machine  back and forth along the floor. With what force does the machine have to be pushed to make it move at velocity  v ? Assume that angular velocity of the disk is large, ω R  ≫  v , and that the force needed to compensate for the torque can be neg- lected. The coefficient of friction between the bristles and the floor is  µ . Here we need fact 5, ideas K5 and 19 and additionally IDEA  26:     These pairs of points are often symmetric- ally located. Idea 20 is relevant as well. PROB  20.  A hexagonal pencil lies on a slope with inclination angle  α ; the angle  between the pencil’s axis and the line of  intersectionoftheslopeandthehorizontal is  ϕ . Under what condition will the pencil not roll down? α ϕ IDEA  27:         What (which vector) could be expressed in terms of its components in our case? The only promising option is the small shift vector of centre of mass when its starts to move; ultimately we are only in- terested in its vertical component. PROB  21.  A slippery cylinder with ra- dius  R  has been tilted to make an angle α  between its axis and the horizontal. A string with length  L  has been attached to the highest point  P  of some cross-section of the cylinder, the other end of it is tied to a weight with mass  m . The string takes its equilibrium position, how long ( l ) is the part not touching the cylinder? The weight is shifted from its equilibrium pos- ition in such a way that the shift vector is parallel to the vertical plane including the cylinder’s axis; what is the period of small oscillations?   l P  IDEA  28:           PROB  22.  A uniform bar with mass  m and length  l  hangs on four identical light wires. The wires have been attached to the  bar at distances  l 3  from one another and are vertical, whereas the bar is horizontal. Initially, tensions are the same in all wires, T  0  =  mg /4. Find tensions after one of the outermost wires has been cut. l l /3 IDEA  29:                 Let us note that this statement is in ac- cordance with idea 3 that gives the num-  ber of available equations (there can be no more unknowns than equations). In this particular case, we are dealing with ef- fectively one-dimensional geometry with no horizontal forces, but the body could rotate (in absence of the wires). Thus we have two degrees of freedom, corres- pondingtoverticalandrotationalmotion. Since the wires are identical, they must have the same stiffness as well; the word “wire” hints at large stiffness, i.e. deform- ations (and the inclination angle of the  bar) are small.   A large proportion of dynamics problems consist of finding the acceleration of some system or forces acting between some  bodies. There are several possible ap- proaches for solving these questions, here we consider three of them. METHOD  4:           x   y    z  We need the same number of equations as we have unknowns; following idea 1 can help to reduce that number. PROB  23.  A block with mass  M  lies on a slippery horizontal surface. On top of it there is another block with mass  m  which in turn is attached to an identical block  by a string. The string has been pulled across a pulley situated at the corner of  the big block and the second small block is hanging vertically. Initially, the system is held at rest. Find the acceleration of the  big block immediately after the system is released. You may neglect friction, as well as masses of the string and the pulley. M   m m This question can be successfully solved using method 4, but we need two more ideas. IDEA  30:        IDEA  31:              Ifbodiesstartatrestorifmotionisalonga straight line, then the same relation holds  between accelerations, since the relation for shifts can be differentiated w.r.t. time. This relation is usually relatively simple,  but in some problems it is easy to make a mistake. METHOD  5:        Method5isusefulinmanyquestionscon- cerning wedges, where it can be difficult to write out the condition for an object to stayonthewedgeinthelaboratoryframe. Applying idea 31 is also often easier in the wedge’s frame of reference than in the laboratory frame. Since the body defining the reference frame is at rest, we can write out the condition(s) of equilibrium for it. FACT  9:        PROB  24.  A wedge has been made out of a very light and slippery material. Its upper surface consists of two slopes making an angle  α  with the horizontal and inclined towards one another. The  block is situated on a horizontal plane; a ball with mass  m  lies at the bottom of the hole on its upper surface. An- other ball with mass  M  is placed higher than the first ball and the system is re- leased. On what condition will the small  ball with mass  m  start slipping upwards along the slope? Friction can be neglected. M  m α α The final method is based on using  gener- alised coordinates  and originates from the- oretical mechanics. There its description requires relatively complicated mathem- atical apparatus, but in most problems it can be used in a much simpler form. METHOD  6:   ξ          ¨ ξ    ξ     Π   Π ( ξ  )    ξ      K   = M ˙ ξ  2 /2   M       ¨ ξ   = − Π ′ ( ξ  ) / M . Here, a dot denotes differentiation w.r.t. time and dash w.r.t. coordinate  ξ  . Indeed, due to conservation of energy  Π ( ξ  ) + M ˙ ξ  2 /2  = Const. Differentiating that w.r.t. time and using the chain rule, we obtain  Π ′ ( ξ  )  ˙ ξ   + M ˙ ξ   ¨ ξ   =  0. We reach the aforementioned formula after divid- ing through by  ˙ ξ  . PROB  25.  A small block with mass  m  lies on a wedge with angle  α  and mass  M . The  block is attached to a rope pulled over a pulleyattachedtothetipofthewedgeand fixed to a horizontal wall (see the figure). Find the acceleration of the wedge. All surfaces are slippery (there is no friction). m α M  a  = ? Full solution of this problem is given in the hints’ section to illustrate method 6 PROB  26.  A wedge with mass  M  and acute angles  α 1  and  α 2  lies on a horizontal surface. A string has been drawn across a pulley situated at the top of the wedge, its ends are tied to blocks with masses  m 1 and  m 2 . What will be the acceleration of  the wedge? There is no friction anywhere. m 2 m 1 α 1  α 2 M  It may seem that there is more than one degree of freedom in this question: the wedge can move and the string can shift w.r.t. the wedge. However, we are saved  by IDEA  32:    x       x    We can use this circumstance to reduce the effective number of degrees of free- dom. In our particular case, the system consists of two components and thus the shift of component can be expressed by that of the other. IDEA  33:    x     X  C  = ∑  x i m i / ∑  m i ,   m i    i     x i      X  C  = ∫   xdm / ∫   dm   dm  =  ρ ( x ,  y ,  z ) dV   PROB  27.  Two slippery horizontal sur- faces form a step. A block with the same height as the step is pushed near the step, and a cylinder with radius  r  is placed on the gap. Both the cylinder and the block have mass  m . Find the normal force  N   between the cylinder and the step at the moment when distance between the block and the step is √  2 r . Initially, the block and the step were very close together and all bodies were at rest. Friction is zero everywhere. Will the cylinder first separ- ate from the block or the step? √  − 2 r m m    r It is easy to end up with very complic- ated expressions when solving this prob- lem, this may lead to mistakes. Therefore it is wise to plan the solution carefully be- fore writing down any equations. IDEA  34:      But how to find acceleration(s) in that case? It is entirely possible if we use method 6, but this path leads to long expressions. A tactical suggestion: if  you see that the solution is getting very complicated technically, take a break and think if there is an easier way. There is a “coincidence” in this particular problem: straight lines drawn from the sphere’s centre to points of touching are perpen- dicular; can this perhaps help? It turns out that it does. IDEA  35:    Let us remind what we learned in kin- ematics: IDEA  K29:      v 2 / R   v     R      ε R    ε    The centre of mass of the cylinder under- goes rotational motion, method 6 is ne- cessary to find angular acceleration — but we hoped to refrain from using it. An im- provement on idea 1 helps us out: IDEA  36:      We can easily find the cylinder’s velocity (and thus the radial component of accel- eration) if we use IDEA  37:             forces changing in time (force acting on a moving point, moving inclined plane) change energy as well. Idea 31 helps to write out conservation of energy (relation  between bodies’ velocities!). To answer the second question, we need IDEA  38:    Also, review idea 31 for horizontal com- ponents of accelerations. PROB  28.  Lightwheelswithradius  R  are attached to a heavy axle. The system rolls alongahorizontalsurfacewhichsuddenly turns into a slope with angle  α . For which angles  α  willthewheelsmovewithoutlift- ing off, i.e. touch the surface at all times? Mass of the wheels can be neglected. The axle is parallel to the boundary between horizontalandslopedsurfacesandhasve- locity  v . m α v IDEA  39:        If normal force has to be negative at that point, then the body lifts off; the critical value is zero — compare with idea 38). Also, review ideas 1, 37 and K29. PROB  29.  A block with mass  M  lies on a horizontal slippery surface and also touches a vertical wall. In the upper sur- face of the block, there is a cavity with the shape of a half-cylinder with radius  r . A small pellet with mass  m  is released at the upper edge of the cavity, on the side closer to the wall. What is the maximum velocity oftheblockduringitssubsequentmotion? Friction can be neglected. r     m    M  IDEA  40:    IDEA  41:      You will also need idea 37. IDEA  42:       0 =  dv dt  =  a       PROB  30.  A light rod with length 3 l  is attached to the ceiling by two strings with equal lengths. Two balls with masses  m and  M  are fixed to the rod, the distance  betweenthemandtheirdistancesfromthe ends of the rod are all equal to  l . Find the tension in the second string right after the first has been cut. m l M  l l There are several good solutions for this problem, all of which share applying idea 34 and the need to find the angu- lar acceleration of the rod. Firstly, angu- lar acceleration of the rod can be found from method 6 by choosing angle of ro- tation  ϕ  to be the generalised coordinate. Secondly, we may use Newton’s 2nd law for rotational motion: we find the torque on the rod about the point of attachment of the second string and equate it to  I  ε with angular acceleration  ε  and moment of inertia  I   =  ml 2 +  4  Ml 2 . More gener- ally, IDEA  43:     s      M  =  I  ε     I      s   I   = ∑  m i r 2 i  = ∫   r 2 · dm  = ∫   r 2  ρ · dV     r i     i    s      K   =  1 2 I  ω 2  Once the angular acceleration is found, in order to apply the idea 34 it may be help- ful to use IDEA  44:     ⃗  F  =  d ⃗  P dt   ⃗  P     ⃗  F      ⃗   M  =  d ⃗  L dt   ⃗  L       ⃗   M    In our case this last method is fruitful when applied both to forces and torques. Another solution method is to con- sider the rod and the balls as three dif- ferent (interacting) bodies. Then the balls’ accelerations can be found as per idea 31; one can also employ IDEA  45:      Clearly if this were not true, a non-zero force would generate an infinite accelera- tion for a massless body. PROB  31.  An inextensible rough thread with mass per unit length  ρ  and length L  is thrown over a pulley such that the length of one hanging end is  l . The pul- ley is comprised of a hoop of mass  m  and radius  R  attached to a horizontal axle by light spokes. The initially motionless sys- tem is let go. Find the force on the axle immediately after the motion begins. The friction between the pulley and the axle is negligible.  R Why not proceed as follows: to find the force, we will use idea 34; the acceleration of the system will be found using Method 6. To apply idea 34 most handily, let us employ IDEA  46:     ⃗  F  =  M ⃗  a C    ⃗  a C    This idea is best utilised when a part of  the system’s mass is motionless and only a relatively small mass is moved about (just like in this case: the only difference after a small period of time is that a short length of thread is “lost” at one end and “gained” at the other end). Obviously idea 32 will be useful here, and idea 19 will save you some effort. Bear in mind that in this case we are not interested in the centre of mass coordinate per se, but only in its change as a function of time; therefore in the expression for this co- ordinate we can omit the terms that are independent of time: their time derivat- iveswillvanish. Thetime-dependentpart of the centre of mass coordinate should be expressed using the same coordinate that we will use with Method 6 (since Method 6 will produce its second derivative with respect to time). A technical bit of ad- vice may help: a vector is specified by  (a) its magnitude and direction;  (b)  its projec- tions onto coordinate axes in a given co- ordinate system; IDEA  47:      Above all, this applies when the direction of the vector is neither known nor appar- ent. In this instance, we should find  F x and  F  y  in a suitable coordinate system. PROB  32.  A thread is thrown over a pul- ley. At its both ends there are two blocks with equal masses. Initially the two blocks are at the same height. One of them is instantaneously given a small horizontal velocity  v . Which of the two blocks will reach higher during the subsequent mo- tion? The pulley’s mass is negligible. v This problem is really tough, because the key to the solution is a very specific and rarely used IDEA  48:      Here the centre of mass can move about a little bit, but in the longer term (aver- aged over one period of the pendulum- like motion of the kicked block — cf. idea 22) it is motionless: the blocks have the same mass and if one of them rises, then in the expression for the centre of mass this will be compensated by the descent of the other block. This is also true for the horizontal coordinate of the centre of  mass, but it is enough to consider the ver- tical coordinate only to solve the problem. Let us also bring up the rather obvious FACT  10:        The solution algorithm is then as follows: we write down Newton’s 2nd law for  (a) the system made out of two blocks and (b)  one block; we average both equations and use the equality apparent from  (a) to find the average tension in the thread, which we then substitute into equation (b) . Based on idea 22, we partition the ten- sion in the threadinto the average and the high-frequency component and use idea 16. PROB  33.  A system of blocks sits on a smooth surface, as shown in the fig- ure. The coefficient of friction between the blocks is  µ , while that between the  blocks and the surface is  µ  =  0. m m x M M   F  The bottom right block is being pulled by a force  F . Find the accelerations of all  blocks. IDEA  49:          For example, if we are to assume that there is no slipping between two touch- ing bodies, then they could be treated as a whole. Then one should find the fric- tional force  F h  between the bodies and de- termine when the assumption holds, or when is  F h  less that the maximum static friction force  µ N  . PROB  34.  A billiard ball hits another sta- tionary billiard ball. At which collection of points could the stationary ball be po- sitioned such that it would be possible to achieve the situation where both balls will fall into two (different) pockets on the table? The collisions are perfectly elastic, the balls are perfectly slippery (hence the rotation of the balls is negligible). IDEA  50:         To prove this, note that the three velocity vectors (velocity before and the two ve- locities after the impact) form a triangle  because of the momentum conservation law. The conservation of energy means that the sides of the triangle satisfy Py- thagore’s theorem. A special case of this result is (see the problem after next) FACT  11:        PROB  35.  An absolutely elastic and slip- pery billiard ball is moving with velocity  v towardtwomotionlessidenticalballs. The motionless balls are touching and their centres lie on a straight line that is per- pendicular to the incoming ball’s velocity vector. The moving ball is directed ex- actly toward the touching point of the two  balls. Which velocity will the incoming  ball have after the collisions? Consider two scenarios:  (a)  the incoming ball hits exactly in the middle between the balls;  (b) its trajectory is a little bit off and it hits one of the stationary balls marginally earlier. v To answer the first question, it is neces- sary to use IDEA  51:              Also, donotforgetidea37! Forthesecond question, let us use IDEA  52:        PROB  36.  n  absolutely elastic beads are sliding along the frictionless wire. What is the maximum possible number of col- lisions? The sizes of the beads are negli- gible, and so is the probability that more than two beads will collide at the same time. IDEA  53:   Here is an auxiliary question: what would the elastic collision of two balls on an  x − t  diagram look like? PROB  37.  A plank of length  L  and mass  M  is lying on a smooth horisontal surface; on its one end lies a small block of mass m . The coefficient of friction between the  block and the plank is  µ . What is the min- imal velocity  v  that needs to be imparted to the plank with a quick shove such that during the subsequent motion the block would slide the whole length of the board andthenwouldfallofftheplank? Thesize of the block is negligible. L µ µ =0  m M  v This problem has two more or less equi- valent solutions. First, we could solve it using idea 6. Second, we could use ideas 37 and 51, further employing IDEA  54:        Indeed, the friction force has a constant magnitude and, as seen in the reference frame of the support, it is always parallel to displacement. PROB  38.  The given figure has been pro- duced off a stroboscopic photograph and it depicts the collision of two balls of equal diameters but different masses. The arrow notes the direction of motion of one of the  balls before the impact. Find the ratio of  the masses of the two balls and show what the direction of motion for the second ball was before the impact. IDEA  55:            To be more specific: when two bodies in- teract, the vector of the impulse is equal tothevectorialdifferenceoftheirtwomo- menta. Cf. idea 5. FACT  12:      FACT  13:      PROB  39.  There are two barrels (  A  and B ) whose taps have different design, see figure. The tap is opened, the height of the water surface from the tap is  H  . What ve- locity does the water stream leave the bar- rels with? H  H  A B IDEA  56:        Itcouldnotbeotherwise: theanswersare, after all, different. It pays to be attentive here. While designing the tap  A , there was a clear attempt to preserve the lam- inarity of the flow: energy is conserved. However, if, motivated by method 3, we were to write down the momentum given to the stream by the air pressure during an infinitesimal time  dt  —  pSdt  (where S  is the tap’s area of cross-section), we would see that, owing to the flow of wa- ter,  p  ̸ =  ρ  g  (cf. dynamical pressure, Bernoulli’s law!). On the other hand, for tap  B  the laminar flow is not preserved; therewillbeeddiesandlossofenergy. We could nonetheless work with momentum: we write the expression for the pressure exerted on the liquid by the walls of the  barrel (generally the pressures exerted by the left and the right hand side walls of  the barrel cancel each other out, but there remains an uncompensated pressure  p  =  ρ  gH   exerted to the left of the cross-section of the tap  S ). PROB  40.  Sand is transported to the con- struction site using a conveyor belt. The length of the belt is  l , the angle with re- spect to the horizontal is  α ; the belt is driven by the lower pulley with radius  R , powered externally. The sand is put onto the belt at a constant rate  µ  (kg/s). What is the minimal required torque needed to transport the sand? What is the ve- locity of the belt at that torque? The coefficient of friction is large enough for the sand grains to stop moving immedi- ately after hitting the belt; take the ini- tial velocity of the sand grains to be zero. α µ  l R FACT  14:      For this problem, idea 56 and methode 3 will come in handy in addition to IDEA  57:          σ  v  =     σ  ( x )     v ( x )  For a flow of incompressible (constant density) liquid in a pipe, such a density is  σ   =  ρ S  and therefore  vS  =  Const. For a region of space where the flow is dis- charged — a sink — the mass increases: dm dt  =  σ  v  — this equation, too, could be called the condition for continuity. PROB  41.  A ductile blob of clay falls against the floor from the height  h  and starts sliding. What is the velocity of the  blob at the very beginning of sliding if  the coefficient of friction between the floor and the blob is  µ ? The initial horizontal velocity of the blob was  u . IDEA  58:         µ  Indeed, ∆  p ⊥  = ∫   N  ( t ) dt  (integrated over the duration of the impact) and  ∆  p ∥  = ∫   µ N  ( t ) dt  =  µ ∫   N  ( t ) dt . PROB  42.  A boy is dragging a sled by the rope behind him as he slowly ascends a hill. What is the work that the boy does to transport the sled to the tip of the hill if its height is  h  and the horizontal dis- tance from the foot of the hill to its tip is a ? Assume that the rope is always parallel to the tangent of the hill’s slope, and that the coefficient of friction between the sled and the snow is  µ .         h a FACT  15:        Clearly, to apply the fact 15, one will need idea 3. PROB  43.  An empty cylinder with mass  M  is rolling without slipping along a slantedsurface, whose angleof inclination is  α  =  45 ◦ . On its inner surface can slide freely a small block of mass  m  =  M /2. What is the angle  β  between the normal to the slanted surface and the straight line segment connecting the centre of the cyl- inder and the block? β  α  m  M  Clearly the simplest solution is based on idea 6, but one needs to calculate the kin- etic energy of a rolling cylinder. IDEA  59:  K   =  K  c  +  M Σ v 2 c /2    K  c      M Σ    ⃗  P  =  M Σ ⃗  v c   ⃗  P c  ≡  0   ⃗  L  =  L c  + ⃗  r c × ⃗  P    I   =  I  0  +  M Σ a 2   I      s    I  0     s     a    We will have to compute angular mo- mentum already in the next problem, so let us clarify things a little. IDEA  60:   ⃗  L  =  ∑  ⃗  L i    i   ⃗  L i  = ⃗  r i × ⃗   p i    L i  =  h i  p i  =  r i  p ti  h i  =  r i  sin α i      p ti  =  p i  sin α      If in a three-dimensional space the an- gular momentum is a vector, for a mo- tion in a plane this vector is perpendicu- lar to the plane and is therefore effectively a scalar (and thus one can abandon cross products). It is often handy to combine ideas 59 and 60: we do not divide the sys- tem into particles but, instead, into rigid  bodies ( L  =  ∑  L i ), we compute the mo- ment of inertia  L i  of each body according to idea 59: the moment of inertia of the centre of mass plus the moment of inertia as measured in the centre of mass frame. IDEA  61:       l   1 12  Ml 2    2 5  MR 2   2 3  MR 2    1 2  MR 2   a    1 6  Ma 2  If the the rotation axis does not go through the centre of mass, then one can (a) findthemomentofinertiawithrespect to the axis of interest using the parallel- axis (Steiner) theorem;  (b)  apply idea 59 to calculate kinetic energy or angular mo- mentum (in which case it is only enough to know the moment of inertia with re- spect to the centre of mass). PROB  44.  A rod of mass  M  and length 2 l is sliding on ice. The velocity of the centre of mass of the rod is  v , the rod’s angu- lar velocity is  ω . At the instant when the centre of mass velocity is perpendicular to the rod itself, it hits a motionless post with an end. What is the velocity of the centre ofmassof therodafter theimpact if  (a) the impact is perfectly inelastic (the end that hits the post stops moving);  (b)  the impact is perfectly elastic. v ω M  2 l In case of an absolutely elastic collision one equation follows from energy conser- vation; if the collision is inelastic, then another condition arises: that of a mo- tionless end of the rod. Still, we have two variables. The second equation arises from IDEA  62:     Indeed, during the impact the body’s motion is affected by the normal and frictional forces, but both are applied through the point of impact: their lever arm is zero. If a body is moving in a gravitational or similar field, then in the longer term the angular momentum with respect to the point of impact may be- gin to change, but immediately before and after the collision it is nonetheless the same(gravityisnottoostrongasopposed to the normal forces that are strong yet short-lived; even though gravity’s lever arm is non-zero, it cannot change the an- gular momentum in an instant). PROB  45.  If one hits something rigid — e.g. a lamppost — with a bat, the hand holding the bat may get stung (hurt) as long as the impact misses the so-called centre of percussion of the bat (and hits either below or above such a centre). De- termine the position of the centre of per- cussion for a bat of uniform density. You may assume that during an impact the bat is rotating around its holding hand. METHOD  7:      Phrased like that, it may seem that the method is rather pointless. However, converting and interpreting real-life scen- arios —  modelling  the problem — is one of  the most challenging and interesting as- pects of physics. It is interesting because it supplies more creative freedom than solving an existing model using well- established ideas. Still, this freedom has limits: the model has to describe the real- ity as best as possible, the approxima- tions have to make sense and it is desir- able that the model were solvable either mentally or with aid of a computer. For a given problem, there is not much free- dom left and the business is simplified: there clear hints as to sensible assump- tions. Let us begin translating: “A rigid rod of length  l  and uniform density is ro- tating around one end with the angular velocity  ω , the rotation axis is perpendic- ular to the rod. At a distance  x  from the axis there is a motionless post that is par- allel to the axis of rotation. The rod hits the post.” Now we encounter the first obstacle: is the impact elastic or inelastic? This is not brought up in the text of the problem. Let us leave it for now: maybe we can get somewhere even without the corresponding assumption (it turns out that this is the case). Now we encounter the central question: what does it mean for the hand “not to get stung”? We know it hurts when something hits our hand — ifthissomethinggetsanimpulsefromthe hand during a short period of time (the impact), as this implies a large force. The hand is stationary, so the hand-held end of the bat should come to halt without re- ceiving any impulse from the hand. Thus our interpretation of the problem is com- plete: “Following the impact, the rotation is reversed, 0  ≥  ω ′  ≥ − ω ; during the impact the axis of rotation imparts no im- pulse on the rod. Find  x .” The penultim- ate sentence hints at the usage of idea 62. PROB  46.  A massive cylinder of radius R  and mass  M  is lying on the floor. A narrow groove of depth  a  has been chis- elled along the circumference of the cylin- der. A thread has been wrapped around the groove and is now being pulled by its free end, held horizontally, with a force  F . The cylinder is positioned such that the thread is being freed from below the cyl- inder. With what acceleration will the cyl- inder start moving? The friction between the floor and the cylinder is large enough for there to be no slipping. F  M   a   R F  ⊙ There are multiple ways to tackle this problem, butletususethefollowingidea. IDEA  63:    I  ε  =  M                To prove this idea, recall idea 6: kinetic energy appears when work is done,  K   = 1 2 I  ω 2 =  M ϕ  ( ϕ  is the angle of rotation of  the body,  ω  =  d ϕ / dt ). If the moment of inertia with respect to the instantan- eous axis of rotation  I   does not depend on time, then  dK  / dt  =  1 2 Id ω 2 / dt  =  I  ωε  = dM ϕ / dt  =  M ω , which gives  I  ε  =  M . PROB  47.  A ball is rolling along a hori- zontal floor in the region  x  <  0 with ve- locity ⃗  v 0  = ( v x 0 , v  y 0 ) . In the region  x  >  0 there is a conveyor belt that moves with velocity ⃗  u  = ( 0, u )  (parallel to its edge  x  = 0). Findthevelocityoftheball ⃗  v  = ( v x , v  y ) with respect to the belt after it has rolled onto the belt. The surface of the conveyor  belt is rough (the ball does not slip) and is level with the floor. IDEA  64:         Indeed, the points where the normal force and the gravity are applied are on the same straight line with the forces them- selves and their sum is zero, meaning that their net torque is also zero; the force of  friction is lying in the plane of the surface, andsoitsleverarmwithrespecttoanaxis in the same plane is zero. PROB  48.  A “spring-dumbbell” com- prises two balls of mass  m  that are con- nect with a spring of stiffness  k  . Two such dumbbells are sliding toward one another, the velocity of either is  v 0 . At some point the distance between them is  L  (see fig.). After which time is the distance between them equal to  L  again? The collisions are perfectly elastic. L v 0  v 0 IDEA  65:                          Note:  this is a rather general idea, division into simpler steps can be useful if rapid (almost instantaneous) processes can oc- cur in a dynamical system; see next prob- lem for an example (also recall idea 51) PROB  49.  Small grains of sand are slid- ing without friction along a cylindrical trough of radius  R  (see fig.). The inclin- ation angle of the trough is  α . All grains have initial velocity zero and start near point  A  (but not necessarily at the point  A  itself). What should be the length of the trough such that all grains would exit it at the point  B ? L α A B IDEA  66:                 2 π   PROB  50.  A coat hanger made of wire with a non-uniform density distribution is oscillating with a small amplitude in the plane of the figure. In the first two cases the longer side of the triangle is ho- rizontal. In all three cases the periods of  oscillation are equal. Find the position of  the centre of mass and the period of oscil- lation. 42cm 10cm Background info:  A finite-size rigid  body that oscillates around a fixed axis is known as the physical pendulum. Its frequency of small oscillations is easy to derive from the relation  I   ¨ ϕ  =  − mgl ϕ , where  I   is the moment of inertia with re- spect to the axis of oscillation and  l  is the distance of the centre of mass from that axis:  ω − 2 =  I  / mgl  =  I  0 / mgl  +  l /  g  (here we employ the parallel-axis/Steiner the- orem, see idea 59). The  reduced length of the physical pendulum is the distance ˜ l  =  l  +  I  0 / ml  such that the frequency of  oscillation of a mathematical pendulum of that length is the same as for the given physical pendulum. IDEA  67:   ˜ l                      Proof: the formula above could be rewrit- ten as a quadratic equation to find the length  l  corresponding to the given fre- quency  ω  (i.e. to the given reduced length ˜ l  =  g / ω 2 ):  l 2 − l ˜ l  +  I  0 / m  =  0. Accord- ing to Vieta’s formulae, the solutions  l 1 and  l 2  satisfy  l 1  +  l 2  =  l , so that  l 1  and l 2  =  ˜ l − l 1  result in the same frequency of  oscillations. PROB  51.  A metallic sphere of radius 2mmanddensity  ρ  =  3000kg/m 3 ismov- ing in water, falling freely with the accel- eration  a 0  =  0,57g. The water density is  ρ 0  =  1000kg/m 3 . With what acceleration would a spherical bubble of radius 1mm rise in the water? Consider the flow to be laminar in both cases; neglect friction. IDEA  68:                Using method 6 we find that in the case (A) the kinetic energy of the system  K   = 1 2 v 2 ( m  +  αρ 0 V  ) , where the constant  α  is a number that characterizes the geometry of the body that correspond to the extent of the region of the liquid that will move (compared to the volume of the body it- self). If a body is acted on by a force  F , then the power produced by this force is P  =  Fv  =  dK  dt  =  va ( m  +  αρ 0 V  ) . Thus F  =  a ( m  +  αρ 0 V  ) : the effective mass of  the body increases by  αρ 0 V  . In the prob- lemabove, theconstant  α  forthespherical  body can be found using the conditions given in the first half of the problem. In case (B), if we assume that the ve- locity of the body is constant, we find K   =  1 2 v 2  ρ 0 ( α Svt ) , where  S  is the cross- sectional area of the body and  α S  is the cross-sectional area of the turbulent ‘tail’. This  α , again, characterizes the body. From here, it is easy to find  Fv  =  dK  dt  = α 2 v 3  ρ 0 S , which gives  F  =  α 2 v 2  ρ 0 S . PROB  52.  A stream of water falls against a trough’s bottom with velocity  v  and splits into smaller streams going to the left and to the right. Find the velocities of   both streams if the incoming stream was inclined at an angle  α  to the trough (and the resultant streams). What is the ratio of  amounts of water carried per unit time in the two outgoing streams? α This is a rather hard problem. Let us first state a few ideas and facts. IDEA  69:    p  +  ρ  gh  +  1 2  ρ v 2 =    p    h     v    FACT  16:      To solve the second half of the problem, the following is needed: IDEA  70:        ⃗  F  =  d ⃗  P dt  + ⃗  Φ P    − ⃗  Φ P     ⃗  Φ P      ⃗  Φ P         The momentum flux of the flowing liquid could be calculated as the product of mo- mentum volume density  ρ ⃗  v  with the flow rate (volume of liquid entering/leaving the system per unit time). What is the open system we should be considering in this case? Clearly, a system that would allow relating the incoming flow rate  µ  (kg/s) to the outgoing fluxes ( µ l  ja  µ r ) using the formula above: a small imaginary region of space that would in- clude the region where the stream splits into two. FACT  17:          PROB  53.  Find the velocity of propaga- tion of small waves in shallow water. The water is considered shallow if the wavelength is considerably larger than the depth of the water  H  . Thanks to this we can assume that along a vertical cross-section the horizontal velocity of all particles  v h  is the same and that the hori- zontal velocity of water particles is signi- ficantly smaller than the vertical velocity. The smallness of the waves means that their height is significantly smaller than the depth of the water. This allows us to assume that the horizontal velocity of  the water particles is significantly smaller than the wave velocity,  u . IDEA  71:                   (An alternative approach is to linearise and solve a system of coupled partial dif- ferential equations.) PROB  54.  A small sphere with mass  m  = 1g is moving along a smooth surface, slid- ing back and forth and colliding elastic- ally with a wall and a block. The mass of  the rectangular block is  M  =  1kg, the ini- tial velocity of the sphere is  v 0  =  10m/s. What is the velocity of the sphere at the instant when the distance between the sphere and the wall has doubled as com- pared with the initial distance? By how many times will the average force (aver- aged over time) exerted by the sphere on the wall have changed? IDEA  72:           I            x    p x  Let us be more precise here. The closed contourisproducedasaparametriccurve (the so-called phase trajectory)  x ( t ) ,  p x ( t ) if we trace the motion of the system dur- ing one full period  T  . The phase traject- ory is normally drawn with an arrow that indicated the direction of motion. The adiabatic invariant is not exactly and per- fectly conserved, but the precision with which it is conserved grows if the ratio τ  / T   grows, where  τ   is the characteristic time of change of the system’s paramet- ers. Adiabatic invariant plays an instru- mental role in physics: from the adiabatic law in gases (compare the result of the previous problem with the adiabatic ex- pansion law for an ideal gas with one de- gree of freedom!) and is applicable even in quantum mechanics (the number of  quanta in the system — e.g. photons — is conserved if the parameters of the sys- tem are varied slowly).   PROB  55.  Astraighthomogeneousrodis  being externally supported against a ver- tical wall such that the angle between the wall and the rod is  α  <  90 ◦ . For which values of   α  can the rod remain stationary whenthussupported? Considertwoscen- arios: a) the wall is slippery and the floor is rough with the friction coefficient  µ  ; b) the floor is slippery and the wall is rough with the friction coefficient  µ . PROB  56.  A light stick rests with one end against a vertical wall and another on a horizontal floor. A bug wants to crawl down the stick, from top to bottom. How should the bug’s acceleration depend on its distance from the top endpoint of the stick? The bug’s mass is  m , the length of  the stick is  l , the angle between the floor and the stick is  α  and the stick’s mass is negligible; both the floor and the wall are slippery ( µ  =  0). How long will it take the  bugtoreachthebottomofthestickhaving started at the top (from rest)? α l   x  a  PROB  57.  A wedge with the angle  α  at the tip is lying on the horizontal floor. There is a hole with smooth walls in the ceiling. A rod has been inserted snugly into that hole, and it can move up and down without friction, while its axis is fixed to be vertical. The rod is suppor- ted against the wedge; the only point with friction is the contact point of the wedge and the rod: the friction coefficient there is µ . For which values of   µ  is it possible to push the wedge through, behind the rod,  by only applying a sufficiently large hori- zontal force? α µ  F  PROB  58.  Sometimes a contraption is used to hang pictures etc. on the wall, whose model will be presented below. Against a fixed vertical surface is an im- movable tilted plane, where the angle  between the surface and the plane is  α . There is a gap between the surface and the plane, where a thin plate could be fit. The plate is positioned tightly against the vertical surface; the coefficient of friction  between them can be considered equal to zero. In the space between the plate and the plane a cylinder of mass  m  can move freely, its axis being horizontal and paral- lel to all considered surfaces. The cylin- der rests on the plate and the plane and the coefficients of friction on those two surfaces are, respectively,  µ 1  and  µ 2 . For whichvaluesofthefrictioncoefficientsthe plate will assuredly not fall down regard- less of its weight? α F  µ  = 0 µ 2 µ 1 m PROB  59.  On top of a cylinder with a horisontal axis a plank is placed, whose length is  l  and thickness is  h . For which radius  R  of the cylinder the horizontal po- sition of the plank is stable? R   l PROB  60.  A vessel in the shape of a cylinder, whose height equals its radius R  and whose cavity is half-spherical, is filled to the brim with water, turned up- side down and positioned on a horizontal surface. The radius of the half-spherical cavity is also  R  and there is a little hole in the vessel’s bottom. From below the edges of the freely lying vessel some water leaks out. How high will the remaining layer of  water be, if the mass of the vessel is  m  and the water density is  ρ ? If necessary, use the formula for the volume of a slice of a sphere (see Fig.):  V   =  π   H  2 ( R −  H  /3 ) . h  H  V   PROB  61.  A vertical cylindrical vessel with radius  R  is rotating around its axis withthe angular velocity  ω . Byhow much does the water surface height at the axis differ from the height next to the vessel’s edges? PROB  62.  A block with mass  M  is on a slippery horizontal surface. A thread ex- tends over one of its corners. The thread is attached to the wall at its one end and to a little block of mass  m , which is inclined by an angle  α  with respect to the vertical, at the other. Initially the thread is stretched and the blocks are held in place. Then the blocks are released. For which ratio of  masses will the angle  α  remain unchanged throughout the subsequent motion? α m M  PROB  63.  Two slippery ( µ  =  0) wedge- shaped inclined surfaces with equal tilt angles  α  are positioned such that their sides are parallel, the inclines are facing each other and there is a little gap in  between (see fig.). On top of the surfaces are positioned a cylinder and a wedge- shaped block, whereas they are resting one against the other and one of the  block’ssidesishorizontal. Themassesare, respectively,  m  and  M . What accelerations willthecylinderandtheblockmovewith? Find the reaction force between them. PROB  64.  Three little cylinders are con- nected with weightless rods, where there is a hinge near the middle cylinder, so that the angle between the rods can change freely. Initially this angle is a right angle. Two of the cylinders have mass  m , another one at the side has the mass 4 m . Find theaccelerationoftheheaviercylinderim- mediately after the motion begins. Ignore friction. 90 o m m 4 m  a =? PROB  65.  A slippery rod is positioned at an angle  α  with respect to the horizon. A little ring of mass  m  can slide along the rod, to which a long thread is attached. A small sphere of size  M  is attached to the thread. Initially the ring is held motion- less, and the thread hangs vertically. Then the ring is released. What is the accelera- tion of the sphere immediately after that? m M  PROB  66.  A block begins sliding at the uppermost point of a spherical surface. Find the height at which it will lose con- tact with the surface. The sphere is held in place and its radius is  R ; there is no fric- tion. PROB  67.  The length of a weightless rod is 2 l . A small sphere of mass  m  is fixed at a distance  x  =  l  from its upper end. The rod rests with its one end against the wall and the other against the floor. The end that rests on the floor is being moved with a constant velocity  v  away from the wall. a)  Find the force with which the sphere affects the rod at the moment, when the angle between the wall and the rod is  α  = 45 ◦  ;  (b)  what is the answer if   x ̸ =  l ? α v m x   2   l    PROB  68.  A light rod with length  l  is connected to the horizontal surface with a hinge; a small sphere of mass  m  is con- nected to the end of the rod. Initially the rod is vertical and the sphere rests against the block of mass  M . The system is left to freely move and after a certain time the block loses contact with the surface of  the block — at the moment when the rod forms an angle  α  =  π  /6 with the hori- zontal. Find the ratio of masses  M / m  and the velocity  u  of the block at the moment of separation. m            l M  PROB  69.  At a distance  l  from the edge of the table lies a block that is connected with a thread to another exact same block. The length of the thread is 2 l  and it is extended around the pulley sitting at the edge of the table. The other block is held above the table such that the string is un- der tension. Then the second block is re- leased. What happens first: does the first  block reach the pulley or does the second one hit the table? l l PROB  70.  A cylindrical ice hockey puck with a uniform thickness and density is given an angular velocity  ω  and a transla- tional velocity  u . What trajectory will the puck follow if the ice is equally slippery everywhere? In which case will it slide farther: when  ω  =  0 or when  ω  ̸ =  0, as- suming that in both cases  u  is the same? PROB  71.  A little sphere of mass  M hangs at the end of a very long thread; to that sphere is, with a weightless rod, attached another little sphere of mass  m . The length of the rod is  l . Initially the system is in equilibrium. What horizontal velocity needs to be given to the bottom sphere for it to ascend the same height with the upper sphere? The sizes of the spheres are negligible compared to the length of the rod. M  m v  l         PROB  72.  A block of mass  m  lies on a slippery horizontal surface. On top of it lies another block of mass  m , and on top of  that — another block of mass  m . A thread that connects the first and the third block has been extended around a weightless pulley. The threads are horizontal and the pulleyisbeingpulledbyaforce  F . Whatis the acceleration of the second block? The coefficient of friction between the blocks is µ . F  a =? m m m PROB  73.  A boy with mass  m  wants to push another boy standing on the ice, whose mass  M  is bigger that his own. To that end, he speeds up, runs toward the other boy and pushed him for as long as they can stand up. What is the maximal distance by which it is possible to push in this fashion? The maximal velocity of a run is  v , the coefficient of friction between  both boys and the ice is  µ . PROB  74.  A uniform rod with length  l  is attached with a weightless thread (whose length is also  l ) to the ceiling at point  A . The bottom end of the rod rests on the slippery floor at point  B , which is exactly  below point  A . The length of   AB  is  H  , l  <  H   <  2 l . The rod begins to slide from rest; find the maximal acceleration of its centre during subsequent motion.    l l         H A B PROB  75.  A stick with uniform density rests with one end against the ground and with the other against the wall. Initially it was vertical and began sliding from rest such that all of the subsequent motion takes place in a plane that is perpendicu- lar to the intersection line of the floor and the wall. What was the angle between the stick and the wall at the moment when the stick lost contact with the wall? Ignore friction. PROB  76.  A log with mass  M  is sliding along the ice while rotating. The velocity of the log’s centre of mass is  v , its angu- lar velocity is  ω . At the moment when the log is perpendicular to the velocity of  its centre of mass, the log hits a station- ery puck with mass  m . For which ratio of the masses  M / m  is the situation, where the log stays in place while the puck slides away, possible? The collisions are per- fectly elastic. The log is straight and its linear density is constant. v ω M  m PROB  77.  A ball falls down from height h , initially the ball’s horizontal velocity was  v 0  and it wasn’t rotating. a) Find the velocity and the angular velocity of the  ball after the following collision against the floor: the ball’s deformation against the floor was absolutely elastic, yet there was friction at the contact surface such that the part of the ball that was in con- tact with the floor stopped. b) Answer the same question with the assumption thatthevelocitiesofthesurfacesincontact never homogenized and that throughout the collision there was friction with coeffi- cient  µ . PROB  78.  A ball is rolling down an in- clined plane. Find the ball’s acceleration. The plane is inclined at an angle  α , the coefficient of friction between the ball and the plane is  µ . PROB  79.  A hoop of mass  M  and radius r  stands on a slippery horizontal surface. There is a thin slippery tunnel inside the hoop, along which a tiny block of mass m  can slide. Initially all the bodies are at rest and the block is at the hoop’s upper- most point. Find the velocity and the ac- celerationofthehoop’scentralpointatthe moment when the angle between the ima- ginary line connecting the hoop’s central point and the block’s position and the ver- tical is  ϕ . ϕ O A    r PROB  80.  A block with mass  m  =  10g is put on a board that has been made such that, when sliding to the left, the coef- ficient of friction  µ 1  =  0,3, while when sliding to the right it is  µ 2  =  0,5. The  board is repeatedly moved left-right ac- cording to the graph  v ( t )  (see fig.). The graph is periodic with period  T   =  0,01s; the velocity  v  of the board is considered t T  T  /2 v 1   m / s positive when directed to the right. Using the graph, find the average velocity that the block will move with. PROB  81.  A water turbine consists of  a large number of paddles that could be considered as light flat boards with length l , that are at one end attached to a rotat- ing axis. The paddles’ free ends are po- sitioned on the surface of an imaginary cylinder that is coaxial with the turbine’s axis. A stream of water with velocity  v and flow rate  µ  (kg/s) is directed on the turbine such that it only hits the edges of  the paddles. Find the maximum possible usable power that could be extracted with such a turbine.  v     l   µ PROB  82.  A flat board is inclined at an angle  α  to the vertical. One of its ends is in the water, the other one is outside the wa- ter. The board is moving with velocity  v with respect to its normal. What is the ve- locity of the water stream directed up the  board? v      u PROB  83.  Amotor-drivenwagonisused to transport a load horizontally by a dis- tance  L . The load is attached to the side of  the wagon by a cable of length  l . Half of  the time the wagon is uniformly acceler- ated, the other half — uniformly deceler- ated. Find the values of the acceleration  a such that, upon reaching the destination, the load will be hanging down motion- lessly. You can assume that  a ≪  g . PROB  84.  A shockwave could be con- sidered as a discontinuous jump of the air pressure from value  p 0  to  p 1 , propagat- ing with speed  c s . Find the speed which will be obtained, when influenced by the shockwave,  (a)  a wedge-shaped block: a prism whose height is  c , whose base is a right triangle with legs  a  and  b  and which is made out of material with density  ρ ;  b) anbodyofanarbitraryshapewithvolume V   and density  ρ . c s    c           b x  p  p 1  p 0 a PROB  85.  A dumbbell consisting of two elastic spheres connected with a thin steel rod is moving parallel to its axis with a velocity  v  toward another exact same spheres. Find the velocity of the dumbbell after a central collision. Is the kinetic en- ergy of the system conserved? v 0   1.  Write out the balance of torques for the contact point  O  of the hoop and the shaft. What is the angle that the tangent to the shaft at point  O  forms with the horizon (given that the wire slips on the shaft)? 2.  Write down the equation for the torques for the cylinder & block system with respect to the contact point of the cylinder and the inclined plane. What angle with respect to the horizon is formed by the tangent to the cylinder constructed at the position of  the little block? 3.  According to the idea 4, consider the system “rod CD  + the mass  m ” as a whole; there are four forces acting on it:  m ⃗   g , ⃗  F , and the tension forces of the rods, ⃗  T   AC  and ⃗  T  BD . The tension forces are the ones which we don’t know and don’t want to know. Accord- ing to the idea 2, these will drop out from the bal- ance of torques acting on the rod  CD  with respect to the intersection point of   AC  and  BD . Indeed, due to the fact 2, the tension force in the rod  AC  is parallel to  AC ; the same applies to the rod  BD . Now, what must be the torque of force  F ? For what direction of  the force will this torque be achieved with the min- imum magnitude? 4.  Thevectorsumoftheforces ⃗  F  and  m ⃗   g  hastocom- pensate the sum of the friction and the normal force ⃗   f   =  ⃗  N   + ⃗  F h , i.e. has to be at an angle arctan µ  with respect to the normal to the plane. Let us draw the force triangle  m ⃗   g  + ⃗   f   + ⃗  F  =  0: the vector  m ⃗   g  can be drawnimmediately(itsdirectionandmagnitudeare known), the direction of  ⃗   f   can be noted by a straight line passing through the terminal point of   m ⃗   g . ⃗  F  has to connect that straight line to the initial point of   m ⃗   g . For which direction is its magnitude minimal? 5.  Go to the reference frame of the inclined surface (invoke Ideas 6 and 7) and use the same method as forproblem4( ⃗  a + ⃗   g  functionsastheeffectivegravity ⃗   g e ). 6.  Usearotatingreferenceframeassociatedwiththe cylinder (where the block is at rest, and the centri- fugal force ⃗   f  t  is constant and pointing downwards). (a)  The terminal point of the net force of gravity and centrifugal force is moving on a circle and has to  be equal to the net force  ⃗   f   of the normal and fric- tional forces. What is the maximum allowed angle  between the vectors  ⃗   f  t  and  ⃗   f   so that there be no slipping? For which direction of   m ⃗   g  is the angle  between the vectors ⃗   f  t  and ⃗   f   maximal?  (b)  There are still only three forces; as long as there is an equilib- rium, these three vectors must form a triangle and hence, must lay on the same plane. According to the idea 9, we’ll depict the force balance in this plane, i.e. in the plane defined by the vectors ⃗   g  and ⃗   f  t . The approach used in part (a) can still be used, but the terminal point of  ⃗   f  t  +  m ⃗   g  draws only an arc of a full circle. Determine the central angle of that arc. De- pending on the arc length, it may happen that the maximal angle between the surface normal (= the direction of   ⃗   f  t ) and  ⃗   f   is achieved at one of the en- dpoints of the arc. 7.  Based on the Fact no. 4, on which line does the intersection point of the frictional forces have to lie? What can be said about the two angles formed by the frictional force vectors and the thread’s direc- tion. Given the Idea no. 1 (the axis is perpendic- ular with the tension in the thread)? Now combine the two conclusions above. Where is the intersection point of the friction force vectors? What is the direc- tion of the cylinder’s velocity vectors at the points where the cylinder rests on the rough band? Where isthecylinder’sinstantaneousrotationaxis(seehow to find it in the kinematics brochure)? What is the velocity vector of the cylinder’s centre point?  (b) Will the equilibrium condition found above be vi- olated if the surface is uniformly rough? 8.  Draw a circle whose diameter is the straight line connecting the points of support. Use Fact no. 6: which curve can the ball move along? Where is the  bottom-most point of this curve? 9.  Consider the torques acting on the rod with re- spect to the hinge. For which angle  α  will the net forceof the normaland frictionalforcespush therod harder against the board? 10.  By how much will the block descend if the thread is extended by  δ ? 11.  Let’s assume that the horizontal component of  the tension in the rope is  T  x . What is the vertical component of the tension next to the ceiling? Next to the weight? Write down the condition for the bal- ance of the forces acting on a) the weight and b) the system of weight & rope (cf. Idea no. 4). 12.  Seeing as  H   ≪  L , clearly the curvature of the rope is small, and the angle between the tangent to the rope and horizon remains everywhere small. From the horizontal force balance for the rope, ex- press the horizontal component of the tension force T  x  as a function of the length  l  (note that while  T  x remains constant over the entire hanging segment of the rope, we’ll need its value at the point  P  separ- ating the hanging and lying segments). Write down the balance of torques acting on the hanging piece of  the rope with respect to the holding hand (according to what has been mentioned above, the arm of the gravity force can be approximated as  l /2). As a res- ult, you should obtain a quadratic equation for the length  l . 13.  Use Idea 8: change into the reference frame of  the rotating hinge.  a)  Following the idea 15, write down the condition of torque balance with respect to the hinge (Idea no. 2) for a small deviation angle ϕ . Whichgeneratesabiggertorque,  m ⃗   g  orthecentri- fugal force? (Note that alternatively, the idea 17 can  be also used to approach this problem).  b)  Follow- ing the idea 17, express the net potential energy for the small deviation angles  ϕ 1  and  ϕ 2  using the en- ergy of the centrifugal force (which resembles elastic force!) and the gravitational force; according to the idea 16, keep only the quadratic terms. You should obtain a quadratic polynomial of two variables,  ϕ 1 and  ϕ 2 . The equilibrium  ϕ 1  =  ϕ 2  =  0 is stable if it corresponds to the potential energy minimum, i,e, if  the polynomial yields positive values for any depar- ture from the equilibrium point; this condition leads to two inequalities. First, upon considering  ϕ 2  =  0 (with  ϕ 2  ̸ =  0) we conclude that the multiplier of   ϕ 2 1 has to be positive. Second, for any  ϕ 2  ̸ =  0, the poly- nomial should be strictly positive, i.e. if we equate this expression to zero and consider it as a quad- ratic equation for  ϕ 1 , there should be no real-valued roots, which means that the discriminant should be negative. 14.  Apply the ideas 15 ja 18 for such a angular po- sition of the beam, for which the magnitude of the  buoyant force doesn’t change (i.e. by assuming a  balance of vertical forces). From Idea no. 2, draw the axis through the centre of mass. While comput- ing the torque of the buoyant force, use Ideas 19, 20; the cross-section of the underwater part of the  beam could be represented as a superposition of a rectangle and two narrow triangles (one of them of  negative mass). 15.  The container & water system is affected by the gravity and the normal reaction force of the ho- rizontal surface on the liquid. Since we know the pressure of the liquid at the base of the container, we can express the mass of the container from the vertical condition for equilibrium. 16.  To compute the first correction using the per- turbation method we use the Fact 49 and the ref- erence system of the block sliding down uniformly and rectilinearly: knowing the magnitude and the direction of the frictional force we can find its com- ponent in  ⃗  w  and ⃗  u  direction. The sign of the latter flips after half a period, and so it cancels out upon averaging. 17.  Let us choose the origin of the vertical  x -axis to  be a point on the surface of the ocean very far from the iron deposit. For the zero reference point of the Earth’s gravitational potential we shall choose  x  =  0 (i.e.  ϕ earth  =  gx ), forthatoftheirondepositweshall take a point at infinity. Then, for the points on the ocean’s surface very far from the iron deposit, the gravitational potential is zero. It remains to find an expressionforthepotentialabovetheirondepositas a function of   x  (using the principle of superposition) and equate it to zero. 18.  Let us employ the reference frame of the plat- form. Let us the consider the balance of torques with respect to the axis of the small disk (then the lever arm of the force exerted by that axis is zero). Let us divide the disk into little pieces of equal size. The frictional forces acting on the pieces are equal  by magnitude and are directed along the linear ve- locities of the points of the disk (in the chosen refer- ence frame). Since the motion of the disk can be rep- resented as a rotation around an instantaneous axis, then concentric circles of frictional force vectors are formed (centred at the instantaneous rotation axis). Clearly, the net torque of these vectors with respect to the disk’s axis is the smaller, the smaller is the circles’ curvature (i.e. the farther the instantaneous rotation axis is): the torque is zero when the instant- aneous rotation axis is at infinity and the concent- ric circles become parallel straight lines. An instant- aneous rotation axis at infinity means that the mo- tion is translational,  ω 3  =  0 (since the linear velocity v  =  ω 3 r  of a given point is finite, but  r  = ∞ ). 19.  The instantaneous axis of rotation is at a dis- tance  r  =  v / ω  from the disk’s axis. Let’s use the same imaginary slicing as in the previous problem. Now compute the component of the net force in the direction of motion. Notice that the frictional forces on the points that are symmetrical with respect to the instantaneous rotation axis balance each other across a whole circular region of radius  R − r . The non-balancedregionisunfortunatelyshapedforcal- culation. Let us imagine extending the "balanced" region up to  R  (the dashed circle in the figure). The part of this extended balanced region, where there is no actual rotating disk underneath (the dark gray crescent in the figure), could be represented as a su- perposition of the two disks, one rotating clockwise and the other – anticlockwise. In that case the clock- wise component partakes in the balancing, whereas the anticlockwise component remains unbalanced. To sum up, two thin crescent-shaped regions remain unbalanced: one corresponds to the the real disk (light gray in the figure), the other — to a disk ro- tating anticlockwise (dark gray); normal to  ⃗  v , the width of these regions is everywhere equal to  r . The net force is the easiest to find by integrating across the crescent-shaped regions using the polar coordin- ate  ϕ :  | d ⃗  F |  =  A · dS , where  dS  is the area of the surface element;  dF x  =  A cos  ϕ dS  =  B cos 2 ϕ d ϕ , F x  = ∫   dF x  =  B ∫   2 π  0  cos 2 ϕ d ϕ . What are the values of the constants  A  and  B ? O r 20.  Consider the unit vector ⃗  τ   directed along the in- finitesimal displacement vector of the centre of the mass at the instant when the pencil begins moving. Let’s express its coordinates in the Cartesian axes ( x ,  y ,  z ) , where  x  is parallel to the pencil and the ( x ,  y ) -plane is parallel to the inclined slope. Using the spatial rotations formulae we represent it in the newcoordinates ( x ′ ,  y ′ ,  z ) ,whicharerotatedwithre- spect to  ( x ,  y ,  z )  around the  z -axis by an angle  ϕ  (so that the axis  x ′  is horizontal). Using the spatial rota- tions formulae we express the vector’s ⃗  τ   vertical co- ordinate  z ′  in the  ( x ′ ,  y ′ ,  z ′ )  coordinate axes, which is obtained from the axes  ( x ′ ,  y ′ ,  z )  by rotating about the  x ′  by the angle  α . 21.  The string connects the two points with the shortest distance along the cylinder’s side; when unfolded, the cylinder is a rectangle. Consider the vertical plane touching the surface of the cylinder that includes the hanging portion of the string. This plane and the cylinder touch along a straight line s . If you imagine unfolding the cylinder, the angle  between the string and the straight line  s  is equal to the cylinder’s inclination angle  α . Given this,  l  is easy to find. When the weight oscillates, the trace of  the string still stays straight on the unfolded cylin- der. Therefore the length of the hanging string (and thus the weight’s potential energy) do not depend in any oscillatory state on whether the surface of  the cylinder is truly cylindrical or is unfolded into a planar vertical surface (as long as the spatial ori- entation of the axis  s  is preserved). 22.  Write down the two equations describing the  balance of force and torques, and then another one that describes the linear relation between the elong- ations of the string:  T  1 − T  2  =  T  2 − T  3 . 23.  Initially only the vertical forces affect the hanging block, therefore the initial displacement vector is also vertical. If the acceleration of the large  block is  a 1 , that of the block on top of it —  a 2  and that of the hanging block —  a 3 , then  a 1  +  a 2  =  a 3 holds. Now we can write down Newton’s 2nd law for each body. The fourth and the final unknown is the tension in the string. 24.  Go to the reference frame of the wedge-block. In the borderline case, the force of inertia’s and grav- ity’s net force on the ball  m  is normal to the left slope (so that the ball stay at rest there). Consider the net forces acting on the balls. Their compon- ents normal to the surface they rest on are ⃗  F ⊥ 1  and ⃗  F ⊥ 2 . These are equal to the normal forces ⃗  N  1  and ⃗  N  2 acting on the balls and therefore have to have equal magnitudes ( F ⊥ 1  =  F ⊥ 2 ) to ensure that the force bal- ance is achieved horizontally for the wedge-block. 25.  Let’s take the displacement  ξ   of the wedge as coordinate describing the system’s position. If the wedge moves by  ξ  , then the block moves the same amount with respect to the wedge, because the rope is unstretchable, and the kinetic energy changes by Π  =  mg ξ   sin α . The velocity of the wedge is  ˙ ξ   and that of the block is 2  ˙ ξ   sin  α 2  (found by adding velocit- ies, where the two vectors  ˙ ξ   are at an angle  α ), there- fore the net kinetic energy  K   =  1 2 ˙ ξ  2 (  M  +  4 m sin 2  α 2  ) . Then we find  Π ′ ( ξ  ) =  mg sin α  and  M  =  M  + 4 m sin 2  α 2 ; their sum gives the answer. 26.  Again, let’s take the wedge’s displacement as the coordinate  ξ  ; if the displacement of the block alongthesurfaceofthewedgeis  η , thenthecentreof  mass being at rest gives  η ( m 1  cos α 1  +  m 2  cos α 2 ) = (  M  +  m 1  +  m 2 ) ξ  . From here one can extract  η  as a function of   ξ  , but to keep the formulae brief it’s  better not to substitute this expression everywhere. The kinetic energies of the block can be found as sums of horizontal [ 1 2 m i (  ˙ ξ  −  ˙ η cos α i ) 2 ] and vertical [ 1 2 m i (  ˙ η sin α i ) 2 ] energies. 27.  When writing down energy conservation, note that the block’s velocity is twice the cylinder’s ve- locity horizontal component and that the latter is equal to the vertical component, too (why?). Project Newton’s 2nd law onto the axis that passes through the top corner of the step and the cylinder’s centre: this axis is perpendicular both to the normal force  between the block and the cylinder and to the cylin- der’s tangential acceleration. Second question: the ratio of two normal forces is constant (why? what is it equal to? Hint: compare the horizontal acceler- ations of the cylinder and the block and remember Newton’s 2nd law), therefore they will be equal to zero at the same instant. 28.  By projecting Newton’s 2nd law on the axis in the direction of the normal force we see that the nor- mal force is the smallest at the bottommost point of  the trajectory’s arch-shaped part. (There, the centri- petal acceleration is the largest, gravitational force’s component along the axis is the smallest). 29.  The energy of the "pellet & block" system is al- ways conserved; momentum will only start to be conserved once the pellet passes the bottommost point. When it arrives there for the second time, the  block’s velocity is maximal (why?). 30.  Let’s apply Idea no. 44 for  ⃗  P : the system’s net momentum is  P  =  ω lm  +  2 ω lM , net force  F  = ( m  +  M )  g − T  . The same using rotational consid- erations: with respect to the leftmost ball’s initial position, the angular momentum is  l ( 2 ω l )  M  (velo- city is 2 ω l , the velocity’s lever arm —  l ); net torque is  ( T   +  Mg ) l . Now, for the formula given in Idea no. 44 we need the angular acceleration  ε  =  ˙ ω . Let’s find it using Method no. 6:  Π  =  l ϕ ( m  +  2  M ) , K   =  1 2  ˙ ϕ 2 l 2 ( m  +  4  M ) . Another solution route: the ratio of accelerations is 1:2; there are four unknowns (two normal forces, acceleration and string tension); equations: three force balances (for either ball and the rod) and one torque balance (wrt the left end- point of the rod). 31.  Method no. 6: for the generalized coordinate ξ   we can use the displacement of the thread’s end- point. Ideas no. 32,20: the change of the system’s CM  y -coordinate is  ξρ h /  M  ( h  — the difference in the heights of the thread’s endpoints,  M  — the net mass of the system; assume that  ξ   ≪  h ). For the x -coordinate it’s 2 ξρ R /  M . 32.  ⟨ T  ( 1  +  cos α ) ⟩  =  2 mg ,  T   =  ⟨ T  ⟩ +  ˜ T  , where | ˜ T  | ≪  T  . Based on the Idea no. 16 we ignore the tiniest term ⟨  ˜ T  α 2 ⟩  and note that ⟨ α 2 ⟩ > 0. 33.  We have to consider two options: either all the  bodies move together, or the rightmost large block moves separately. Why cannot the situations occur where  (a)  all three components move separately, or (b)  the left large block moves separately? 34.  After the collision the ball’s trajectories are or- thogonal crossing straight lines; the angle with re- spect to the initial trajectory is determined by how much the collision was off-centre. 35.  For slightly non-central motion: what will be the direction of momentum of the ball that was first to be hit? Now apply the Idea no. 50 again. Cent- ral motion: express the velocities after the collision via the horizontal component of the momentum  p x that has been transferred to one of the balls. What is the transferred vertical component  p  y ? Energy con- servation provides us an equation to find  p  y  (it is convenient to express the energy as  p 2 /2 m ). 36.  Thegraphlookslike  n intersectingstraightlines; the intersection point of a pair of straight lines cor- responds to a collision of two balls (the graph of  either ball’s motion is a jagged line; at a collision point the angles of the two jagged lines touch one another so that it looks as if the two straight lines intersect). 37.  Initial velocities in the centre of mass:  mv m +  M ,  Mv m +  M , final velocities are zero; friction does work: µ mgL . 38.  Based on the figure we immediately obtain (to within a multiplicative constant) the magnitudes and directions of the momenta, but not which mo- mentum is which ball’s. It is necessary to find out where the ball marked with an arrow will proceed after the collision. Fact no. 13 will help choose from the three options. 39.  Energy: in time  dt  the distribution of the liquid will change: there is still some water at the centre,  butacertainmass dm hasbeendisplacedfromabove to the level of the tap (and then through the tap), so the change in the system’s potential energy is  gH  · dm . Momentum: the water in the barrel obtains the total momentum  ρ  gHS · dt  from the walls. This momentum is passed on to the stream of water with the mass  ρ Sv · dt . 40.  Energy is not conserved: the grains of sand slip and experience friction. In time  dt  the sand landing on the conveyor belt receives momentum dp  =  v · dm  =  v µ · dt  fromthebelt: theforcebetween the freshly fallen sand and the belt is  F 1  =  dp / dt . The sand already lying on the belt experiences the gravitational force  mg  which is compensated by the component of the friction parallel to the belt,  F 2  = mg cos α , where  m  =  σ  L  is the mass of the sand on the belt and  σ  v  =  µ . The minimization has to be done over  v . 41.  During the collision ∆  p ⊥  = √  2  gh . 42.  Consider a short section of the path along the hill with length  dl . In addition to the change in the potential energy work is done to overcome friction, dA h  =  µ mg tan α · dl . WE find  dA h  =  C · dx , where C  is a constant. Summing over all such little path increments  dl  we find  A h  =  C ∆ x . 43.  The kinetic energy  K   =  m 2  ˙ x 2 +  M  ˙ x 2 , where  x is the displacement along the slanted surface;  Π  = (  M  +  m ) sin α . Having found the acceleration  a  we change into a reference frame (of the cylinder) mov- ing with acceleration  a  (Ideas no. 6 and 7), where the  block is being displaced along the effective accelera- tion due to gravity — as low as possible. 44.  According to the Ideas no. 59 and 60, the angular momentum of the rod before the collision is  L 0  =  Mlv −  1 3  Ml 2 ω ; after the collision  L 1  =  Mlv ′ −  1 3  Ml 2 ω ′ ;  L 1  =  L 2 . The expression for energy is  K   =  1 2  Mv 2 +  1 6  Ml 2 ω 2 . The condition for being at the end:  v ′  + l ω ′  =  0 (we consider  ω  to be positive if  the rotation is in the direction marked in the figure). 45.  The angular momentum with respect to the impact point before the collision:  mv ( x −  l 2 ) − I  0 ω , where  v  =  ω  l 2  and  I  0  =  1 12 ml 2 . 46.  The instantaneous rotation axis passes the con- tact point of the cylinder and the floor; its distance from the centre of mass does not change, so we can use Idea no. 63;  I   =  3 2 mR 2 . 47.  Let us direct the  z  axis upward (this will fix the signs of the angular momenta). The final moment of  inertia with respect to the  x -axis is − 7 5 mv  y R − muR and with respect to the  y -axis is  7 5 mv x R . 48.  Immediately after the first collision the centres of masses of both dumbbells are at rest, the velocit- ies of the colliding balls reverse direction, the non- colliding balls’ velocities don’t change. Both dumb-  bells act like pendula and complete half an oscilla- tion period, after which the second collision occurs – analogous to the first one. 49.  The grains of sand perform harmonic oscilla- tions in the plane perpendicular to the cylinder’s axis — like a mathematical pendulum of length  l  = R  in the gravitational field  g cos α ; along the axis thereisuniformacceleration( a  =  g sin α ). Focussing occurs if the time to cross the trough along its axis is an integer multiple of the oscillation’s half-period. 50.  Observing the equilibrium position we con- clude that the centre of mass lies on the symmetry axis of the hanger. The three suspension points must  be located on the two concentric circles mentioned  by Idea no. 67. Therefore one of the circles must accommodate at least two points out of the three, whilethecircles’centre(thehanger’scentreofmass) must lie inside the region bounded by the hanger’s wires on its symmetry axis. There is only one pair of  circles that satisfies all these conditions. Computing the radii  l 1  and  l 2  of the circles using trigonometry we determine the reduced length of the pendulum l 1  +  l 2  and, using that, the oscillation period. 51.  The effective mass of the moving water can be found using the acceleration of the falling ball. For the rising bubble the effective mass is exactly the same, the mass of the gas, compared to that, is neg- ligibly small. 52.  The water stream could be mentally divided into two parts: the leftmost stream will turn to the left upon touching the trough, the rightmost — to the right. Thus, two imaginary ’water tubes’ form. In either tube the static pressure is equal to the ex- ternal pressure (since there is the liquid’s outer sur- face in the vicinity): according to Bernoulli’s law, the velocity of the liquid cannot change. Based on the conservation of momentum horizontally, the mo- mentum flows of the left- and right-flowing streams have to add up to the original stream’s momentum flow’s horizontal component. Note that due to con- tinuity,  µ  =  µ v  +  µ  p . 53.  Due to continuity  ( u  +  v )(  H   +  h ) =  Hu  Const, where  h  =  h ( x )  is the height of the water at point x  and  v  =  v ( x )  is the velocity. We can write down Bernoulli’s law for an imaginary ’tube’ near the sur- face (the region between the free surface and the stream lines not far from the surface):  1 2  ρ ( u  +  v ) 2 +  ρ  g (  H   +  h ) =  1 2  ρ u 2 +  ρ  gH   =  Const. We can ig- nore that small second order terms (which include the factors  v 2 or  vh ) 54.  The phase trajectory is a horizontal rectangle with sides  L  and 2 mv , where  L  is the distance from the block to the wall; the adiabatic invariant is thus 4 Lmv . 55.  Consider the balance of torques. For the net force vectors of the normal and frictional forces, when you extend them, their crossing point must be above the centre of mass. 56.  Let’s write down Newton’s 2nd law for rota- tional motion with respect to the crossing point of  the normal forces: the angular momentum of the  bugis  L  =  mvl  sin α cos α , thespeedofchangeofthis angular momentum will be equal to the torque due to gravity acting on the bug (the other forces’ lever arms are zero). When computing the period, note that the acceleration is negative and proportional to the distance from the bottom endpoint, i.e. we are dealing with harmonic oscillations. 57.  The blocking occurs if the net force of normal and frictional forces pulls the rod downwards. 58.  Once the blocking occurs we can ignore all the forces apart from normal and frictional ones. Suppose it has occurred. Then the net frictional and normal forces acting from the left and from the right have to balance each other both as forces and torques, i.e. lie on the same straight line and have equal magnitudes. Thus we obtain the angle  between the surface normal and the net force of fric- tion and normal force. 59.  Consider the direction of the torque acting on the plank with respect to the point of contact, when theplankhasturnedbyanangle  ϕ : thecontactpoint shifts by  R ϕ , the horizontal coordinate of the centre of mass shifts by the distance  h 2  ϕ  from the original position of the contact point. 60.  The only force from the surface on the sys- tem vessel & water is equal to the hydrostatic pres- sure  ρ  gh π  R 2 ; it balances the gravitational force  ( m +  ρ V  )  g . Note that  H   =  R − h . 61.  The gravitational potential of the centrifugal force is  1 2 ω 2 r 2 , where  r  is the distance from the ro- tation axis. 62.  Assume the reference frame of the large block (which moves with acceleration  a ). Where does the effective gravity (the net force of the gravity and the force of inertia) have to be directed? What is  a ? With which acceleration does the little block fall in this reference frame? What is the tension  T   of the thread? Having answers to these questions we can write down the equilibrium condition for the large  block  ma  =  T  ( 1 − sin α ) . 63.  Let us use the displacement of the sphere (down the inclined surface) as the generalized coordinate  ξ  . What is the displacement of the sphere (up the other inclined surface)? Evidently  Π  = ( m −  M )  g ξ   sin α . The normal force between the two bodies can be found by projecting Newton’s second law onto the inclined surface’s direction. 64.  Let the displacement of the large cylinder be  ξ  , the horizontal displacement of the middle and the leftmost cylinder, respectively,  x  and  y . What is the relationship between them given that the centre of  mass is at rest? What is the relationship between them given that the length of the rods does not change? From the two equations thus obtained we can express  x  and  y  via  ξ  . If we assume the displace- ment to be tiny, what is the relationship between the vertical displacement  z  of the middle cylinder and the horizontal projection of the rod’s length,  ξ  − x ? Knowing these results, applying Method no. 6 is straightforward. 65.  Where is the small displacement  ξ   of the sphere directed (see Idea no. 30)? What is the displacement of the ring expressed via  ξ  ? Use Method no. 6. 66.  Use Idea no. 38 along with energy conserva- tion by projecting the force and the acceleration in the Newton’s 2nd law radially. 67.  Let us use some ideas from kinematics to find the acceleration of the sphere (K1, K29 and K2: by changing into the reference frame moving with ve- locity  v  we find the component of the sphere’s accel- eration along the rod and by noticing that the hori- zontal acceleration of the sphere is zero, we obtain, using trigonometry, the magnitude of the accelera- tion). Now use Newton’s 2nd law. 68.  Using the velocity  v  of the sphere we can ex- press the velocity of the block at the moment being investigated (bearing in mind that their horizontal velocities are equal). Using Idea no. 38 we find that the block’s (and thus the sphere’s) horizontal accel- eration is zero; by using Newton’s 2nd law for the sphereandthehorizontaldirectionweconcludethat the tension in the rod is also zero. From the energy conservation law we express  v 2 and from Newton’s 2nd law for the sphere and the axis directed along the rod we obtain an equation wherein hides the solution. 69.  UsingNewton’s2ndlawinvestigatewhitherthe system’s centre of mass will move — to the left or to the right (if the centre of mass had not move, then the both events would have happened at the same time). 70.  To answer the first part: show that the force per- pendicular to velocity is zero (use Method no. 3 and Idea no. 26). To answer the second part use Method no. 3 and idea 54. 71.  Due to the length of the thread there are no ho- rizontal forces, i.e. the horizontal component of mo- mentum is conserved, and so is the energy. From the two corresponding equation the limiting velo- city  v  =  v 0  can be found, for which the bottom sphere ascends exactly to the height of the top one. Note that at that point its vertical velocity is zero, cf. Idea no. 42. 72.  Use Idea no. 49. Options: all block keep to- gether; everything slides; the top one slides and and the bottom two stay together (why is it not possible that the top two keep together and the bottom one slides?). 73.  Which conservation law acts when the two boys collide (during a limited time of collision) — do we consider the collision absolutely elastic or inelastic (can momentum be lost and where? If it is inelastic, where does the energy go?), see Idea no. 56? After the collision: the common acceleration of the two  boys is constant, knowing the initial and final velo- cities finding the distance becomes an easy kinemat- ics problem. 74.  Prove that for a vertical thread the velocity is maximal (by applying Idea no. 42 for the rotation angleoftherodshowthatitsangularvelocityiszero in that position; use Idea no. 59). Then it only re- mains to apply energy conservation (remember that ω  =  0). 75.  Find the instantaneous rotation axis (make sure that its distance from the centre of mass is  1 2 ). Prove that the centre of mass moves along a circle centred at the corned of the wall and the floor, whereas the polar coordinate of the centre of mass on that circle is the same as the angle  ϕ  between the wall and the stick. Express the kinetic energy as a function of  the derivative ˙ ϕ  of the generalized coordinate  ϕ  us- ing the parallel-axis (Steiner’s) theorem and express the energy conservation law as  ω 2 =  f  ( ϕ ) ; using Method no. 6 we obtain  ε  =  ˙ ω  =  1 2  f  ′ ( ϕ ) . When the normal force against the wall reaches zero, the acceleration of the centre of mass is vertical: present this condition using the tangential and radial accel- erations of the centre of mass on its circular orbit (  l 2 ε and  l 2 ω 2 respectively) and use it as an equation to find  ϕ . 76.  Based on Idea no. 62 we find that  ω  =  6 v / l . Us- ing energy and momentum conservation we elimin- ate the puck’s velocity after the collision and express the mass ratio. 77.  The forces along the normal to the surface are elastic forces, so the energy in vertical direction is conserved during the collision: after the collision the corresponding velocity component is the same as before. To find the other two unknowns, the ho- rizontal and angular velocities, we can obtain one equation using Idea no. 62. The second equation arises from  (a)  the condition that the velocity of the  ball’s surface is zero at the contact point (no sliding; (b)  the equation arising from 58). 78.  Using the idea 49 we investigate the sliding and rolling regimes. In the latter case the quickest way to find the answer is to use Idea no. 63. 79.  The velocity can be found from the conservation laws for energy and momentum (note that the hoop is moving translationally). To find the acceleration it is convenient to use the non-inertial reference frame of the hoop, where the centripetal acceleration of the  block is easily found. The condition for the radial  balance of the block gives the normal force between the block and the hoop (don’t forget the force of in- ertia!); the horizontal balance condition for the hoop provides an equation for finding the acceleration. 80.  Let us assume the block’s velocity to be ap- proximately constant. For a certain time  t l  the base slides to the left with respect to the block and the momentum imparted by the frictional force at that time is also directed to the left. During the remain- ing time  t r  the base slides to the right with respect- ive momentum directed to the right as well. The equilibrium condition is that the two momenta have equal magnitudes; hence we ding the equilibrium value of   t l / t r . From the graph we find the velocity for which that ratio has the needed value. 81.  As the water flows against the paddles it ob- tainthesameverticalvelocity  u  asthepaddlesthem- selves. This allows to compute the momentum im- parted to the paddle per unit time (i.e. the force), which ends up being proportional to the difference: F  ∝  v − u . From there, it is not very hard to find the maximum of the power  Fu . 82.  In the reference frame of the board the problem is equivalent to the problem no. 52. 83.  Go into the (accelerated) reference frame of the wagon, where the effective gravity √  a 2 +  g 2 is at a smallanglewithrespecttothevertical. Theloadwill oscillate yet remain motionless at the end if the cable is vertical at the stopping moment and the load’s ve- locity is zero. It is possible when the corresponding position is the maximal deviation during the oscil- lation. Therefore the oscillation amplitude has to be the same both during the acceleration and decelera- tion, so that even when the deceleration begins the cable has to be vertical. In that case, how are the ac- celeration time and the oscillation period related? 84.  If the shockwave is at the point where the in- tersection area of its wavefront and the considered  body is  S , then what is the force acting on the body? Let us assume that the body stays (almost) at the same place as the shockwave passes it. Then the mo- mentum imparted during the time  dt  can be found using the cross-sectional area  S  and the distance dx  =  c s  · dt  covered by the wavefront. Note that S · dx  is the volume element. Finally we sum over all imparted momenta. 85.  The rod will act like a spring (since the rod is thin and made out of steel, while steel is elastic). After the left sphere has collided with the station- ary sphere, the latter will acquire velocity  v 0  and the former will stay at rest. Then the dumbbell, as a system of spheres and springs, will begin oscillat- ing around its centre of mass. What is the velocity of  the centre of mass? Convince yourself that after half  a period the single sphere is already far enough that the left sphere is not going to collide with it again. The oscillations of the dumbbell will decay little by little — so some energy will be lost there.   1.  arcsin  R µ ( R + l ) √  µ 2 + 1 . 2.  arcsin  m  M + m µ √  µ 2 + 1 . 3.  mg /2. 4.  a)  µ mg / √  1  +  µ 2 ; b)  mg sin ( arctan µ − α ) . 5.  µ ≥  |  g sin α − a cos α |  g cos α + a sin α  , if   g  +  a tan α > 0. 6.  a)  ω 2 R ≥  g √  1  +  µ − 2 ;  b)  ω 2 R ≥  g √  1  +  µ − 2 , if   µ < cot α  and ω 2 R ≥  g ( cos α  +  µ − 1 sin α )  if   µ > cot α 7.  v /2. 8.  tan2 α  =  h / a 9.  µ 1  ≥ √  l 2 − h 2 / h 10.  3 mg 11.  2arctan [( 1  +  m  M ) cot α ] 12. √  2  HL µ  +  µ 2  H  2 − µ  H   ≈ √  2  HL µ − µ  H   ≈ 7,2m. 13.  a)  ω 2 <  g / l  ; b)  ( 2 − √  2 )  g / l 14.  1 2 ( 1 − 3 − 1/2 )  ρ v  ≈ 211kg/m 3 15.  π  3  ρ R 3 16.  v / 󲈚  µ 2 cot 2 α − 1 17.  4 3 π  Gr 3 ∆  ρ /  g ( r  +  h ) ≈ 0,95cm 18. − ω 19.  µ mgv / ω R 20.  cos  ϕ tan α < tan30 ◦ 21.  L − π  R /2cos α ; 2 π  √  L /  g 22.  1 12 mg ,  1 3 mg ,  7 12 mg 23.  mg / ( 2  M  +  m ) 24.  m <  M cos2 α . 25.  mg sin α / [  M  +  2 m ( 1 − cos α )] = mg sin α / [  M  +  4 m sin 2  α 2 ] . 26.  g  ( m 1  sin α 1 − m 2  sin α 2 )( m 1  cos α 1 + m 2  cos α 2 ) ( m 1 + m 2 +  M )( m 1 + m 2 ) − ( m 1  cos α 1 + m 2  cos α 2 ) 2 . 27.  mg ( 5 √  2 − 4 ) /6 ) ; Simultaneously. 28.  cos α ≥  1 3 ( 2  +  v 2 /  gR ) 29.  2  m  M + m √  2  gR 30.  mMg / ( m  +  4  M ) 31.  F x  =  2 Ra  ρ ,  F  y  =  ρ ( m  +  ρ L )  g − ( L − π  R − 2 l ) a , where  a  =  ρ  g ( L − π  R − 2 l ) / ( m  +  ρ L ) . 32.  The one that had not been pushed. 33.  If   F  ≤  2 µ mg  m +  M 2 m +  M :  a 1  =  a 2  =  1 2 F  M + m ; otherwise  a 1  =  F  M  − µ  g  m  M ,  a 2  =  µ  g  m 2 m +  M . 34.  On a half-circle. 35.  (a) v /5;  (b) v /4. 36.  n ( n − 1 ) /2 37. 󲈚  2 µ  gL ( 1  +  m  M ) 38.  3,5; was coming from below right. 39.  A: √  2  gh  ; √   gh . 40.  2 R µ √   gl  sin α , √   gl  sin α . 41.  u − µ √  2  gh . 42.  mg ( h  +  µ a ) . 43.  arctan  2 5  ≈ 21 ◦ 48 ′ . 44.  (a)  ( ω l  +  3 v ) /4;  (b)  ( ω l  +  v ) /2. 45.  At a distance 2 l /3 from the holding hand, where  l  is the length of the bat. 46.  2 3 F  M a R 47.  ( v x 0 , v  y 0 − 5 7 u ) 48.  L / v 0  +  π  √  m /2 k  49.  1 2 π  2 ( n  +  1 2 ) 2 R tan α 50.  1,03s 51.  2,0g 52.  v 1  =  v 2  =  v ; cot 2  α 2 53. √   gH  . 54.  5m/s. 55.  (a)  tan ≤ 2 µ  ;  (a)  impossible. 56.  g ( 1 −  x l  ) sin − 1 α ;  π  2 √  l  sin α /  g 57.  µ < cot α . 58.  µ 1  < tan  α 2  and  µ 2  < tan  α 2 . 59.  R > h /2 60.  3 √  3 m / πρ 61.  ω 2 R 2 /2  g 62.  M / m  =  cot α − 1. 63.  2 mM  M + m  g tan α 64.  g /9. 65.  g  m +  M m +  M sin 2 α  sin 2 α . 66.  2/3 R 67.  m [  g − v 2 ( 2 l − x ) / √  2 l 2 ] 68.  M / m  =  4,  u  = √   gl /8. 69.  The first one arrives first 70.  A straight line; if   ω ̸ =  0 71. √  2  gl ( 1  +  m /  M ) 72.  F 3 m , if   F m µ  g  < 6;  F 4 m  +  1 2 µ  g , if  6 <  F m µ  g  < 10; 3 µ  g , if   F m µ  g  > 10 73.  m 2 v 2 /2 (  M 2 − m 2 ) µ  g 74. 󲈚  ( l −  H  2  )  g 75.  arccos  2 3  ≈ 48 ◦ 12 ′ 76.  M / m  =  4. 77.  (a)  ω  =  5 v 0 /7 R ,  v x  =  5 v 0 /7,  v  y  = √  2  gh ; (b) v  y  = √  2  gh ,  v x  =  v 0 − 2 µ v  y , ω  =  5 √  2  gh µ / R . 78.  5 7  g sin α , if   µ  >  2 7  tan α , otherwise  g sin α − µ  g cos α 79. 􂈚   2  gr m +  M 1 + cos  ϕ m sin 2 ϕ +  M m cos  ϕ ;  gm sin2 ϕ m sin 2 ϕ +  M [ 1 2  +  m 2 cos  ϕ ( 1 + cos  ϕ ) ( m sin 2 ϕ +  M )( m +  M ) ] 80.  0,6m/s 81.  1 4 µ v 2 82.  v /cos α 83.  n − 2 Lg /4 π  2 l ,  n  =  1,2,... 84.  (a),(b)  (  p 1 −  p 0 ) V  / mc s . 85.  1 2 v 0 ; no, a fraction goes into the longit- udinal oscillations of the rod and then (as the oscillations die) into heat